LSAT and Law School Admissions Forum

Get expert LSAT preparation and law school admissions advice from PowerScore Test Preparation.

Lesson help relating to our Advanced Logical Reasoning Course.
 T.B.Justin
  • Posts: 194
  • Joined: Jun 01, 2018
|
#61574
I am wondering about the incorrect answer choice (C):

Even if the language used was, "Some of the paintings by Renoir and Cezanne that are owned by the Federici Art Museum were purchased by Ms. Federici herself."

How would that relate to the conclusion of the boards action to the quality of the museum collection?

(I mentally noted this as a loser because I think the author wasn't relying on this as an assumption since I found it to be unrelated to the conclusion)
User avatar
 Dave Killoran
PowerScore Staff
  • PowerScore Staff
  • Posts: 5850
  • Joined: Mar 25, 2011
|
#61747
Hey TB,

If we assume that "collected for" is the same as "purchased by," then the modified "some" statement you mention would be something that is known to be true from the sentence that reads, "among the paintings that the late Ms. Federici collected for the museum were several unsuccessful immature works by Renoir and Cezanne." So, that would count as an assumption and thus as a correct answer (since the argument relates to it in some way). However, they aren't likely to do something like that since it comes directly from one of the statements in the stimulus!

The question of whether "collected for" is the same as "purchased by" is actually pretty interesting, and although it doesn't figure in this problem, I'd suspect that the two overlap but aren't seen as identical since "collecting" could include trading for art or discovering a piece somehow.

Anyway, please let me know if that helps. Thanks!
 T.B.Justin
  • Posts: 194
  • Joined: Jun 01, 2018
|
#61759
However, they aren't likely to do something like that since it comes directly from one of the statements in the stimulus!
This brought up PT 62, Sec 2, #25

Where an assumption question, the correct answer choice (a): the proposed explanation for the decline in applications applies in this case.

I bring this up because I was not expecting to see this in an assumption question and it is stated in that stimulus.

Thanks Dave

Get the most out of your LSAT Prep Plus subscription.

Analyze and track your performance with our Testing and Analytics Package.